Search found 207 matches


Thanks a lot. got it now :)

by The Jock

Thu Dec 02, 2010 12:58 am
Forum: Problem Solving
Topic: Combinations 3
Replies: 9
Views: 1502

Let the rate of the reaction be R Let concentration of chemical A be A Let concentration of chemical B be B Then R is proportional to A² R is also proportional to 1/B Hence, R is proportional to A²/B If C is a constant, R=C*(A²/B) If the concentration of B is increased 100% B becomes 2B ( B+(100/...

by The Jock

Thu Dec 02, 2010 12:54 am
Forum: Problem Solving
Topic: don't understand the question..
Replies: 6
Views: 1455

This is a combination problem involving a restriction, so recall that: total number of selections = restricted selections + permitted selections or permitted selections = total number of selections - restricted selections The total number of selections is just all the ways we can pull 3 people out ...

by The Jock

Thu Dec 02, 2010 12:27 am
Forum: Problem Solving
Topic: Combinations 3
Replies: 9
Views: 1502

The answer should be A . Conclusion:Abolition of government regulation of airfares has increased competition among airlines and thus will eventually lead to compromises in airline safety But what if consumer select airlines based on safety parameter only and if safety records are not good for one co...

by The Jock

Wed Dec 01, 2010 12:09 am
Forum: Critical Reasoning
Topic: CR TC 55 7
Replies: 5
Views: 3099

I think that it should be D.
First bold face is saying that experimental design has withstood all serious challenges: Assertion that undermine the hypothesis that magnetic monopoles must exist. because if the design is good then it should have found the MM.
Second bold face is the hypothesis.

by The Jock

Tue Nov 30, 2010 11:58 pm
Forum: Critical Reasoning
Topic: Magnetic Monopoles
Replies: 6
Views: 2542
by The Jock

Tue Nov 30, 2010 6:50 pm
Forum: Critical Reasoning
Topic: Healthcare, always an issue
Replies: 46
Views: 20257

Seems B to me.
Conclusion: The principal should be fired.
weaken question, so anything giving an alternate choice for school's debacle should weaken.
B is the one that is is not impacting anything here....

by The Jock

Tue Nov 30, 2010 6:47 pm
Forum: Critical Reasoning
Topic: You're fired!
Replies: 40
Views: 17944

Hey David, I think the answer is E . My Reasoning: within past six months women have become majority of workforce and these past six months were hit by recession. in the recession time firms are firing manufacturing workers and those without degrees but hiring those who are better educated. So if fi...

by The Jock

Mon Nov 29, 2010 6:31 pm
Forum: Critical Reasoning
Topic: Women in the workforce
Replies: 16
Views: 3402

What is the approximate likelihood that 2 people will choose same number?

Is it 6/256, if yes then I can explain.

by The Jock

Fri Nov 26, 2010 12:49 am
Forum: Problem Solving
Topic: help,help, help, combination hardest,
Replies: 6
Views: 1174

Brutal SC: A group of students

A group of students who have begun to clean up Frederick Law Olmstead's Morning Park in New York City believes that the park needs not to be redesigned but to be returned to its former condition . a)believes that the park needs not to be redesigned but to b)believe that the park needs to not be rede...

by The Jock

Thu Nov 25, 2010 6:19 pm
Forum: Sentence Correction
Topic: Brutal SC: A group of students
Replies: 17
Views: 3436

OA is B.Thanks Rishab for nice explanation.

by The Jock

Thu Nov 25, 2010 5:54 pm
Forum: Critical Reasoning
Topic: LSAT: Identical Twins.....Assumption question
Replies: 9
Views: 4334

LSAT: Identical Twins.....Assumption question

The brains of identical twins are genetically identical. When only one of a pair of identical twins is a schizophrenic, certain areas of the affected twin‟s brain are smaller than corresponding areas in the brain of the unaffected twin. No such differences are found when neither twin is schizophre...

by The Jock

Thu Nov 25, 2010 10:39 am
Forum: Critical Reasoning
Topic: LSAT: Identical Twins.....Assumption question
Replies: 9
Views: 4334

yeah Rishabh, you are right. Please explain.

by The Jock

Thu Nov 25, 2010 10:23 am
Forum: Critical Reasoning
Topic: LSAT: Flaw in reasoning
Replies: 4
Views: 2441

LSAT: Flaw in reasoning

Smith: Meat in the diet is healthy, despite what some people say. After all, most doctors do eat meat, and who knows more about health than doctors do? Which one of the following is a flaw in Smith‟s reasoning? (A) Attacking the opponents‟ motives instead of their argument (B) Generalizing on th...

by The Jock

Thu Nov 25, 2010 9:54 am
Forum: Critical Reasoning
Topic: LSAT: Flaw in reasoning
Replies: 4
Views: 2441

D should be right answer here. If I am not wrong "That" relative pronoun can be used for Singular as we Plural. The condition itself do not make tree grow, so A is out. Conditions help the tree to grow so we need that after conditions.

by The Jock

Wed Nov 24, 2010 9:28 pm
Forum: Sentence Correction
Topic: The Growing Trees
Replies: 6
Views: 2031